Help with Setting Up/Simplifying Euler-Lagrangian

  • Thread starter Thread starter stephenklein
  • Start date Start date
Click For Summary
The discussion focuses on simplifying the Euler-Lagrangian equations related to a string of length l in the xy-plane. The user successfully derived partial derivatives for part (c) but is skeptical about the simplification of their resulting equation. They reached a form that includes a sum and an additional term, y'', which differs from the expected answer. The conversation highlights the importance of clarity regarding which area is to be maximized, as the problem statement lacks specific details. Overall, the user is seeking confirmation and guidance on their approach to the Euler-Lagrangian problem.
stephenklein
Messages
6
Reaction score
3
Homework Statement
Imagine we have a string of length ##l## in the ##x y## plane with endpoints ##x=-a## and ##x=a##. The endpoints can move to maximize this area, but the total length of the string is fixed.

a) Show that ##dx = \sqrt {1-y'^2}## where ##ds## is a tiny length of the curve such that ##ds = \sqrt {dx^2+dy^2}## as in class.

b) The area of the shaded rectangle above is ##ydx##, so the sum of all of those areas between ##x=-a## and ##x=a## gives us the total area. However, it’s easier to incorporate the fact that the length is fixed as ##l## by integrating with respect to s, the path length of moving along the string instead. Using the expression in part (a) to convert the ##ydx## integral to an integral with respect to ##ds##, express the area under the string.

c) Using the Euler-Lagrange equations prove that the optimal shape of the string is a semicircle.

A couple hints:

1) The Euler-Lagrange equation should yield ##\frac {dy} {ds} = \sqrt {1- \frac {y^2} {C^2}}##, which you can solve using separation of variables (and probably looking up the integral).

2) A semicircle of radius R has the equation ##x^2+y^2=R##
Relevant Equations
Euler-Lagrange equation: $$\frac {\partial f} {\partial y} - \frac {d} {ds} \frac {\partial f} {\partial y'} = 0$$
I was able to work through parts (a) and (b). For part (c), I got $$\frac {\partial f} {\partial y} = \sqrt {1-y'^2}$$ and $$\frac {\partial f} {\partial y'} = \frac {-y y'} {\sqrt {1-y'^2}}$$ Taking ##\frac {d} {ds}## of the latter, I used the product rule for all three terms ##y, y', (\sqrt{1-y'^2})^{-1/2}## and my result was $$\sqrt {1-y'^2} + \frac {y y''+y'^2} {\sqrt {1-y'^2}} + \frac {y y'^2 y''} {(1-y'^2)^{3/2}} = 0$$ I'm unsure (even more, skeptical) that this result simplifies to the one given in the question. I'm confident in everything up until the last derivative, which has a lot of moving parts. Any thoughts?

EDIT: I simplified the above result to ##y'=\sqrt {1+y y''}##, which agrees with a Wolfram widget I found that simplifies Lagrangian equations. I feel like I'm very close, but my result has a sum instead of a difference, and that pesky ##y''## term isn't in the given answer.
 
Last edited:
Physics news on Phys.org
stephenklein said:
Homework Statement: Imagine we have a string of length ##l## in the ##x y## plane with endpoints ##x=-a## and ##x=a##. The endpoints can move to maximize this area
What area? You have only told us about a string of length ##l##. Which area is to be maximised?
 
Question: A clock's minute hand has length 4 and its hour hand has length 3. What is the distance between the tips at the moment when it is increasing most rapidly?(Putnam Exam Question) Answer: Making assumption that both the hands moves at constant angular velocities, the answer is ## \sqrt{7} .## But don't you think this assumption is somewhat doubtful and wrong?

Similar threads

  • · Replies 4 ·
Replies
4
Views
2K
Replies
4
Views
2K
  • · Replies 6 ·
Replies
6
Views
2K
Replies
6
Views
2K
  • · Replies 2 ·
Replies
2
Views
1K
Replies
2
Views
1K
  • · Replies 18 ·
Replies
18
Views
3K
  • · Replies 5 ·
Replies
5
Views
1K
  • · Replies 13 ·
Replies
13
Views
1K
  • · Replies 2 ·
Replies
2
Views
2K